Diffeomorphismen und Randbedingungen

Ich versuche herauszufinden, wie die Autoren in diesem Artikel ( arXiv:0809.4266 ) die allgemeine Form des Diffeomorphismus herausgefunden haben, die die Randbedingungen in demselben Artikel beibehalten.

Ich habe dieses Papier gefunden ( arXiv: 1007.1031 ), das das durch Lösen sagt L ξ G μ v , für Komponenten und das Gleichsetzen jeder Komponente mit der entsprechenden Randbedingung, kann ich die allgemeinste erhalten ξ (was schließlich mein Ziel ist).

Also habe ich die Metrik Near-Horizon Extremal Kerr (NHEK) genommen, die 6 nicht verschwindende Terme hat ( G τ φ = G φ τ das gibt mir also 5 zu lösende Gleichungen), ich setze die Randbedingungen ( Ö ( R N ) Begriffe), und um die Dinge etwas zu vereinfachen, habe ich alles in Mathematica eingegeben. Aber als ich meine 5 Differentialgleichungen einfügte, bekam ich den Fehler, dass ich zu viele Gleichungen und zu wenige Variablen habe ( τ , R , θ , φ )!

Jetzt dachte ich, muss ich alles Mögliche einbeziehen G μ v ? Nun, das würde nicht viel Sinn machen, da alle anderen Terme der Hintergrundmetrik Null sind, oder? Und selbst wenn ich sie einfüge, bekomme ich mehr Gleichungen und immer noch nur 4 Variablen :\ Also wird Mathematica wahrscheinlich den gleichen Fehler geben ...

Habe ich also zuallererst recht, wenn ich versuche, den Diffeomorphismus auf diese Weise zu finden? Und wenn ich richtig liege, wie löst man das?! Es ist ein großes System von ODEs, und es ist nicht so trivial zu lösen, wenn man bedenkt, wie die Metrik aussieht :\

Wenn Sie also einen Vorschlag haben, würde ich mich darüber freuen ...

Ich habe das auf math.stackexchange gepostet, weil ich hier keine Antwort gefunden hatte ...

Antworten (1)

Der schwierigste Teil besteht darin, überhaupt einen Satz konsistenter Randbedingungen zu erhalten – dies erfordert eine Kombination aus fundiertem Raten, physikalischen Erkenntnissen, vorheriger Erfahrung mit verwandten Problemen, detaillierten Berechnungen und Trial-and-Error. Kurz gesagt, es ist ein bisschen eine Kunst.

Sobald Sie jedoch eine Reihe von Randbedingungen haben (wie in Ihrem Fall die NHEK-Randbedingungen), sind die Dinge ziemlich einfach.

Lassen Sie mich die asymptotische Hintergrundmetrik mit bezeichnen G und die zustandsabhängigen Schwankungen um H , sodass jede Metrik des Formulars G + Ö ( H ) ist durch die Randbedingungen erlaubt.

Ihr Ziel ist es, zu überprüfen, welche Spurweitentransformationen Ihre Randbedingungen beibehalten. In reiner Schwerkraft müssen dies einige Diffeomorphismen sein, die von einem Vektorfeld erzeugt werden ξ , so dass

L ξ ( G + H ) = Ö ( H )
Wo L ξ ist die Lie-Ableitung. Da dies eine Gleichung für einen symmetrischen Tensor in ist D Maße, die Sie erhalten D ( D + 1 ) / 2 unabhängige lineare PDEs erster Ordnung für das Vektorfeld ξ .

Wenn Sie versucht haben, die obige Gleichung zu lösen, haben Sie genau das Richtige getan, was hoffentlich einen Teil Ihrer Frage beantwortet. Lassen Sie mich nun auf den anderen Teil eingehen, nämlich wie man diese PDEs löst.

In vielen Beispielen können Sie nach dem allgemeinsten Vektorfeld auflösen ξ kompatibel mit obiger Bedingung, indem man einfach einen passenden Ansatz errät und dann zeigt, dass es funktioniert.

In den meisten Anwendungen haben Sie eine Reihenerweiterung der asymptotischen Metrik in Potenzen einer radialen Koordinate R (oder in Exponentialen von R , hängt von Ihrer Wahl des Messgeräts für die radiale Koordinate ab).

Um Unordnung zu vermeiden, lassen Sie mich annehmen, dass die verschiedenen Tensorkomponenten von G Und H werden in einigen Laurent-Reihen ausgedrückt R , und das R entspricht der asymptotischen Grenze.

Dann machen Sie einfach den gleichen Potenzreihenansatz für das Vektorfeld ξ . Typischerweise beginnen alle Komponenten des Vektorfelds bei Ö ( 1 ) oder kleiner, aber das muss nicht der Fall sein. Wenn du überhaupt keine Ahnung hast, dann mach einfach den Ansatz

ξ 0 = R N 0 ( ξ 0 0 + ξ 1 0 / R + . . . )
wo der Koeffizient funktioniert ξ ich 0 von allen Randkoordinaten a priori abhängen dürfen. Einen ähnlichen Ansatz machst du für alle anderen Komponenten des Vektorfeldes.

Die Auswertung der Bedingungen aus der Lie-Variation bestimmt dann die Exponenten N ich und kann die Funktionen einschränken ξ J ich .

Siehe zum Beispiel Übung (17.1) in den Übungen der Woche 7 auf meiner Lehrwebseite http://quark.itp.tuwien.ac.at/~grumil/teaching.shtml für eine Führung durch das Standard-AdS 3 Beispiel. Wenn Sie diese Art von Berechnung noch nie durchgeführt haben, empfehle ich Ihnen, mit diesem Beispiel zu beginnen, bevor Sie den NHEK brechen.

[Nebenbei bemerkt habe ich nie versucht, diesen Algorithmus in Mathematica zu implementieren, da ich normalerweise in 3 Dimensionen arbeite, wo eine Berechnung von Hand ziemlich schnell geht, aber ich sehe keinen Grund, warum es in Mathematica nicht funktionieren sollte.]

Vielen Dank für die ausführliche Antwort! :) Ich werde dieses Problem in meiner Masterarbeit bearbeiten und habe versucht, die Ergebnisse zu erhalten A D S 3 Metrik, die ich im Web gefunden habe, indem ich die Brown-Henneaux-Randbedingungen angewendet habe, aber ich hatte nicht viel Zeit, sie durchzuarbeiten, da ich andere Prüfungen hatte und an einem Seminar arbeitete, aber ich werde Versuchen Sie auf jeden Fall, das herauszufinden, bevor Sie mit dem NHEK beginnen. Nochmals vielen Dank, dass Sie mich in die richtige Richtung geführt haben :)
Hallo, ich versuche, das von Ihnen genannte Problem in Ihren Übungen zu lösen, aber ich glaube nicht, dass ich das richtig mache. Erstens scheine ich nicht zu wissen, wo das ist Ö ( 1 ) ergibt sich aus der Berechnung der Lie-Ableitung der ++-Komponente. Sicher, ich bekomme, wo ich das bekomme 2 G + + ξ , das ist einfach, aber dann die Ö ( 1 ) erscheint :\ Plus wenn ich das unterscheide ξ würde ich nicht die dritte Ableitung von bekommen ε + ( X + ) ? Gibt es Literatur oder gelöste Aufgaben, die ich sehen könnte? Das würde sehr helfen...
Es gibt verschiedene Ö ( 1 ) Beiträge in der ++-Komponente der Lie-Ableitung - zB der erste Term ξ μ μ G + + = Ö ( 1 ) . Vielleicht verstehe ich Ihre Frage im Kommentar nicht - Sie können mir eine E-Mail senden, wenn Sie möchten, da ich den Physik-Stackexchange nicht zu häufig überprüfe. Übrigens taucht der dritte Ableitungsterm in der nächsten Aufgabe (17.2) auf, spielt aber keine wichtige Rolle für die randbedingungserhaltenden Eichtransformationen.